You are on page 1of 211

4

5
Example: proton 1 has a speed v0 (v0<<c) and is moving along the x-axis in the +x
direction. Proton 2 has the same speed and is moving parallel to the x-axis in the –
x direction, at a distance r directly above the x-axis. Determine the electric and
magnetic forces on proton 2 at the instant the protons pass closest to each other.
This is example 28.1 in your text. y
The electric force is
 2 FE
 1 q1q2 ˆ 1 e ˆj v0
FE = r FE = 2 2
4   r 2 4   r
B1
To calculate the magnetic force: at the position of proton 2 there is a magnetic
1r̂
r
field due to proton
 1.
    ev 0 ˆ
  q1 v1  rˆ B = k E
B1 = 1 2 FB 1 v
4 r 2 4  r =   v 2 x
 ˆ
 ev 0 i  jˆ   F
  0 0

B1 =  FB = q2 v 2  B1 E

4 r2 z
  2 2

 
ˆ  
FB = ev 0  i  

 4 r
ev
2
0 ˆ
k  FB =

 

4 r
e v0 ˆ
2
j
Both forces are in the +y direction. The ratio of their
magnitudes is
   e2 v 20 
 2 
FB  4  r 
=
FE  1 e2 
 2  y
 4   r 
FE
FB v0 FB
=     v 20 2
FE
B1 r
Later we will find that r̂
1
v0 x
1
 = 2
c
z
FB v 20
Thus = 2
FE c

If v0=106 m/s, then


FB
=
10 6 2

 1.11  10 -5

FE 3 10 8 2 y

FE
v0 FB
2

B1 r

1
v0 x

z
11
12
(22
14
15
Biot–Savart Law

Where 0 is the permeability of free space

18
19
20
B of a stright wire
 
  0 I  r0  0 I sin 
dB  dl ; dB  dy
4 r 2
4 r 2

x
r  x  y , sin  
2 2 2

x2  y2
 0 Ix a
dy  0 Ia
B 
4  a ( x  y )
2 2 3/ 2

2x( x 2  a 2 )1/ 2
a
dy 2a
a ( x 2  y 2 )3 / 2  x 2 ( x 2  a 2 )1/ 2
Long wire: a>>x
0 I
B
2x
22
23
25
26
27
28
29
30
31
example
Two infinitely long wires form the x and y axis of a 2D coordinate system.
They each carry current as shown. What is the magnetic field at a point
P (2.0 m, 2.0 m)?
A. 710−7 T into the page I1 = 3.0 A
 P
B. 710−7 T out of the page
C. 110−7 T into the page I2 = 4.0 A

D. 110−7 T out of the page


E. 0

0 3 A Btotal  B1  B2
B1  into the page
2 2 m 

0  4 A  0 1 A
B2  out of the page Btotal   10 7 T out of the page
2 2 m  2 2 m 
32
33
34
35
The net magnetic field
In which direction is the net magnetic field at the origin in
the situation shown below? All the wires are the
same distance from the origin.

1. Left
2. Right
3. Up
4. Down
5. Into the page
6. Out of the page
7. The net field is zero
The net magnetic field
We add the individual fields to find the net field, which is
directed right.
November 7, 2007
39
40
41
42
43
44
46
47
48
49
50
51
Magnetic field of a magnetic dipole
A circular loop or a coil currying electrical current is a magnetic dipole, with magnetic
dipole moment of magnitude =NiA.
Since the coil curries a current, it produces a magnetic field, that can be calculated
using Biot-Savart’s law:

  
0  0 
B( z )  
2 ( R  z )
2 2 3/ 2
2 z 3
All loops in the figure have radius r or 2r. Which of
these arrangements produce the largest magnetic
field at the point indicated?
53
Magnetic Field of a current loop
Magnetic field produced by a wire can be enhanced
by having the wire in a loop. x1

1 loop Current  I

B I

N loops Current  NI
x2

54
Examples:
FIND B FOR A POINT LINED UP WITH A SHORT STRAIGHT WIRE

i ds P  0
i ds  r̂  0 dB  2
i ds sin()  0
r̂ 4r
Find B AT CENTER OF A HALF LOOP, RADIUS = r

 0i  0i
B   
4r 4r
into page
Find B AT CENTER OF TWO HALF LOOPS

 0i  0i
OPPOSITE B  2x  
CURRENTS 4r 2r
same as closed loop

 0i  0i
PARALLEL B  -  0
CURRENTS 4r 4r
into out of
page page
Copyright R. Janow – Spring 2014
What is the magnitude and ]4[
direction of the magnetic field at
?.point Q in Fig

I
Bloop  int o
2R
I
Bwire  out
2R
1 I
Bnet  (1  )  8.6 10 7 T int o
 2R

56
57
November 7, 2007
November 7, 2007
November 7, 2007
November 7, 2007
November 7, 2007
[10] The segment of wire in Figure carries a current of I =5.00 A, where the
radius of the circular arc is R = 3.00 cm. Determine the magnitude and
direction of the magnetic field at the origin.

The quarter circle makes one forth the field

a full loop

1 0 I
B ( )  26.2 T int o
4 2R

63
November 7, 2007
Example: Determine the magnetic field (in terms of I, a, and d) at the
origin due to the current loop in Figure P30.17. The loop extends to
infinity above the figure.

November 7, 2007
November 7, 2007
November 7, 2007
November 7, 2007
November 7, 2007
November 7, 2007
Magnetic Force Between Two Parallel Conductors

0 I 2
B2 
2a
0 I 2
F1  B2 I1l  I1l
2a

F1  0 I1 I 2

l 2a
Magnetic Force Between Two Parallel Conductors
• The force (per unit length) on wire 1
due to the current in wire 1 and the
magnetic field produced by wire 2:
FB  0 I1 I 2

l 2a
• Parallel conductors carrying
currents in the same direction
attract each other

• Parallel conductors carrying


currents in the opposite directions
repel each other
November 7, 2007
A loop and a wire
A loop with a clockwise current is placed below a long
straight wire carrying a current to the right. In which
direction is the net force exerted by the wire on the
loop?

1. Left
2. Right
3. Up
4. Down
5. Into the page
6. Out of the page
7. The net force is zero
A loop and a wire
The forces on the left and right sides cancel, but the forces on
the top and bottom only partly cancel – the net force is
directed up, toward the long straight wire.

I1

a
I2

b
L

0 I1I 2 L  1 1 
Fnet  I 2 L( Btop  Bbottom )    
2  a a  b 
Example
Suspending a current with a current. A horizontal wire
carries a current I1=80A dc. A second parallel wire 20cm
below it must carry how much current I2 so that it doesn’t
fall due to the gravity? The lower has a mass of 0.12g per
meter of length.
Which direction is the gravitational force? Downward
This force must be balanced by the magnetic force exerted on the wire by
the first wire. Fg mg FM 0 I1 I 2
  
l l l 2 d
mg 2 d
Solving for I2 I2  
l 0 I1
  
2 9.8 m s 2  0.12  10 3 kg  0.20m 
 15 A
 4  10 7

T  m A  80 A 
77
Which direction should the current flow? The same direction as I1.
78
79
November 7, 2007
November 7, 2007
November 7, 2007
November 7, 2007
November 7, 2007
November 7, 2007
Example
A long straight wire of radius R carries a current I that is uniformly distributed
over the circular cross section of the wire. Find the magnetic field both
outside the wire and inside the wire.

Let’s look at the OUTSIDE field, ro > R


I
B  dl   0 I enc
B(2r )   o I
o I
Boutside  ri
2r ro

R
Example
A long straight wire of radius R carries a current I that is uniformly distributed over the circular
cross section of the wire. Find the magnetic field both outside the wire and inside the wire.

Let’s look at the INSIDE field, ri < R

We first need to identify exactly what is


I
the ENCLOSED current. It isn’t , “I”, but
rather a FRACTION of “I”.

I enc I r2
 2 I enc I 2
r R
2
R
B  dl   0 I enc Since the current is
distributed throughout ri
ro
the cross section we
r2
B(2r )   o I 2 can set up a ratio of
the currents as it
R

R
relates to the cross
o Ir sectional area.
Binside 
2R 2
Example
How could the magnetic field be graphically displayed?
o I 1
Boutside  B
2 r r I
o Ir
Binside  B r
2 R 2

ri
ri ro
ro
R
 

C
B  d s   0 i
r
Current into
page, field
tangent to the
B (2r )   0ienclosed closed
amperian loop
 0ienclosed
B
2r 2
i r
ienclosed  J (r )  2 r  i 2
2 2

R R
 0ir
B For r < R
2R 2
Coaxial cable 1.r  a
 0 Ir
B
2R 2
2.a  r  b
0 I
B
2r
3.b  r  c
B 2r   0 I   0 J (r 2  b 2 )
I
J
 (c 2  b 2 )
0 I r 2  b2
B (1  2 )
2r c b 2

4.r  c, B  0 Self-shielding
Into
ttp://
webdirectory.hcmiu.
du.vn/Portals/25/
serfiles/4627/
Ngoc/Lecture13.pdf

95
96
Solenoids
 We saw earlier that a complete loop of
wire has a magnetic field at its center:
 0i
B
2R
 We can make the field stronger by
simply adding more loops. A many
turn coil of wire with current is called a
solenoid.
 We can use Ampere’s Law to calculate
B inside the solenoid.

 The field near the wires is still circular,


but farther away the fields blend into a
nearly constant field down the axis.

November 7, 2007
November 7, 2007
Solenoid Magnetic Field
• Now let’s use Ampere’s law to determine the magnetic
field inside a very long, densely packed solenoid

B=0

• Let’s choose the path abcd, far away from the ends
– We can consider four segments of the loop for integral
b   c  d   a 
–  B  dl  a B  dl  b B  dl  c B  dl d B  dl
– The field outside the solenoid is negligible. So the integral on
ab is 0.
– Now the field B is perpendicular to the bc and da segments. So
these integrals become 0, also.
99
Solenoid Magnetic Field
– So the sum becomes:
  d  
 B  dl  
c
B  dl  Bl

– If a current I flows in the wire of the solenoid, the total current


enclosed by the closed path is NI
• Where N is the number of loops (or turns of the coil) enclosed
– Thus Ampere’s law gives us Bl  0 NI
– If we let n=N/l be the number of loops per unit length, the
magnitude of the magnetic field within the solenoid becomes
– B   nI 0
• B depends on the number of loops per unit length, n, and the current
• But does not depend on the position within the solenoid but uniform inside
it, very similar to a bar of magnet
100
Solenoids
 The actual field looks more like this:
 Compare with electric field in a capacitor.
 Like a capacitor, the field is uniform inside
(except near the ends), but the direction
of the field is different.
 Approximate that the field is constant inside
and zero outside (just like capacitor).
 Characterize the windings in terms of
number of turns per unit length, n. Each
turn carries current i, so total current over
length h is inh.
 
 B  ds  Bh  0ienc  0inh
B   0in ideal solenoid only section that has non-zero
contribution
November 7, 2007
November 7, 2007
November 7, 2007
Toroid: A long solenoid bent into a circle
Find the magnitude of B field inside
i outside  Draw an Amperian loop parallel to the field, with
flows up radius r (inside the toroid)
 The toroid has a total of N turns
 The Amperian loop encloses current Ni.
LINES OF  B is constant on the Amperian path.
CONSTANT  
B ARE
CIRCLES  B  ds  B  2r   0ienc   0iN
 0iN
B inside toroid
2 r
• N times the result for a long thin wire
• Depends on r
• Also same result as for long solenoid
N
n (turns/unit length)  B   0in
2 r

Find B field outside


AMPERIAN LOOP IS Answer B0 outside
A CIRCLE ALONG B
Toroids
 Notice that the field of the solenoid sticks out
both ends, and spreads apart (weakens) at the
ends.
 We can wrap our coil around like a doughnut, so
that it has no ends. This is called a toroid.
 Now the field has no ends, but wraps uniformly
around in a circle.
 What is B inside? We draw an Amperian loop
parallel to the field, with radius r. If the coil has
a total of N turns, then the Amperian loop
encloses
 current Ni.

 B  ds  B 2r  0ienc  0iN
 0iN
B inside toroid
2 r
November 7, 2007
November 7, 2007
B Outside a Toroid
 0iN
5. B
The magnetic field inside a Toroid is .
2 r
Using an Amperian loop, what is the expression
for the magnetic field outside?
A. Zero
B. The same, decreasing as 1/r.
C. The same, except decreasing as
1/r2.
D. The same, except increase as r.
E. Cannot determine.

November 7, 2007
November 7, 2007
November 7, 2007
November 7, 2007
November 7, 2007
November 7, 2007
November 7, 2007
November 7, 2007
November 7, 2007
November 7, 2007
November 7, 2007
November 7, 2007
November 7, 2007
Example
Two wires, each having a weight per units length of 1.0x10-4
N/m, are strung parallel to one another above the surface of
the Earth, one directly above the other. The wires are
aligned north-south. When their distance of separation is
0.10 mm what must be the current in each in order for the
lower wire to levitate the upper wire. (Assume the two wires
.carry the same current)
l
1
I1
2 d
I2

120
F1
1
B2 I1
mg/l
2 d
I2
l

121
122
123
124
:Example
Consider a solenoid consisting of 100 turns of wire and
length of 10.0 cm. Find the magnetic field inside when it
.carries a current of 0.500 A

125
What is the net force on the ]1[
? .rectangular loop of wire in Fig

The force on the top and bottom of the loop


.are equal and opposite so they cancel

1A  2m    1A
Fright   4  10 7 N to the left
2  1m

1A  2m    1A
Fleft   8  10 7 N to the right
2  0.5m

Fnet  4  10 7 right

126
What is the net force per ]2[
unit length on the wire
? .carrying 2A in Fig

The 2A wire is attracted to the 1A wire (force up) and attracted to the 3A
wire ( force down). Thus, the force per unit length on the 2A wire is

F  0 (2 A)(1A)  0 (2 A)(3 A)
 up  down
l 2 (1m) 2 (1m)
 8  10 7 N / m down

127
What is the magnitude and direction ]3[
? .of the magnetic field at point P in Fig

 0 (1A)
B1   2 107 T int o
2 (1m)
 0 (2 A)
B2   4 107 T out
2 (1m)
 0 (3 A)
B3   3  107 T out
2 (2m)
 (4 A)
B4  0  4 107 T out
2 (2m)
BT   B1  B2  B3  B4  0.9 106 T out

128
A positive charge q = 3.20 × 10-19 C moves with a velocity ]8[
v = (2iˆ + 3jˆ - kˆ ) m/s through a region where both a uniform
.magnetic field and a uniform electric field exist
Calculate the total force on the moving charge (in unit-vector notation), taking B = (2iˆ + 4jˆ )a(
+ kˆ ) T and E =(4iˆ - jˆ - 2kˆ ) V/m. (b) What angle does the force vector make with the
?positive x axis

a ] Lorentz force is given by

F  Felectric  Fmagnetic
F  qE  q (v  B )  q ( E  v  B )
find ( E  v  B ) ??

F  (3.52i  1.6 j )  10 18 N


Fy
b]   tan 1
 24.4
Fx

129
Figure shows a section of a long hallow cylindrical of radii a=5cm
and b=10 cm, carrying a uniform distributed current I, the
magnitude of the magnetic field on its outer surface at r=b is
measured to be B=0.2T, where r is the radial distance from the
.cylindrical axis
?Find the current in the wire )a(

Amper ' s Law on the outer surface (r  b)

 B.dl   i0 enc

B(2b)   0i
2Bb 2  0.2  0.1
i   10 5
A
0 4 10 7

130
?Find the magnitude of the magnetic field at r=20 cm )b(

 B.dl   i 0 enc

B (2r )   0i
 0i 4 10 10 7 5
B   0.1T
2r 2  0.2

131
?Find the magnitude of the magnetic field at r=8 cm )c(

 B.dl   i
0 enc

r 2  a 2
B (2r )   0 ( 2 )i
b  a 2

 0i r 2  a 2
B ( 2 )
2 r (b  a ) 2

4 10 7  105 0.082  0.052


B ( )  013T
2 0.08(0.10  0.05 )
2 2

132
133
Faraday’s Law

134
:Question
On the previous chapters we were calculating the magnetic
.field due to current carrying conductors
:But Now
Can a magnetic field produce an electric field that can
?induce an electric current

135
Electromagnetic Induction

• emf is induced in a conductor placed in a magnetic


field whenever there is a change in magnetic field.
Michael Faraday
1791 – 1867
•British physicist & chemist
•Great experimental scientist
Contributions to Electricity:
•He was the first to produce an electric
current from a magnetic field, invented
the first electric motor and dynamo.

Copyright © 2009 Pearson Education, Inc.


Faraday’s Experiment – Set Up
•A primary coil is connected
to a switch and a battery.

•No battery is present in the


secondary coil.

•The secondary coil is not directly


connected to the primary coil.

Copyright © 2009 Pearson Education, Inc.


Close the switch
& observe the
current readings
on the ammeter.

Copyright © 2009 Pearson Education, Inc.


Faraday’s Findings
•At the instant the switch is closed,
the ammeter changes from zero in one
direction, then returns to zero.
•When the switch is opened,
the ammeter changes in the opposite
direction, then returns to zero.
•The ammeter reads zero when there is a
steady current or when there is no
current in the primary circuit.
Copyright © 2009 Pearson Education, Inc.
Faraday’s Experiment: Conclusions
•An electric current can be induced in a loop
by a changing magnetic field.
–This would be the current in the secondary
circuit of this experimental set-up.
•The induced current exists only while the
magnetic field through the loop is changing.
•This is generally expressed as:
An induced emf is produced in the loop
by the changing magnetic field.
•Just the existence of the magnetic flux is not sufficient to
produce the induced emf, the flux must be changing.
Copyright © 2009 Pearson Education, Inc.
•He found no evidence when the current was steady. He saw
an induced current when the switch was turned on or off.

He concluded:
A Changing Magnetic Field Induces an EMF.
•His experiment used a magnetic field that was changing because
the current producing it was changing; the picture shows a
magnetic field that changes because the magnet is moving.
Copyright © 2009 Pearson Education, Inc.
Induced Current, Summary

Copyright © 2009 Pearson Education, Inc.


Faraday’s Law of Induction: Lenz’s Law
•Faraday found that the induced emf in a wire loop is
Proportional to the time Rate of Change of
the
Magnetic Flux Through the Loop.
•Magnetic Flux is defined similarly to electric flux:

•If B is constant over the surface area A,


then
ΦB = BA = BA cosθ
(The scalar or dot product of vectors B & A)
Copyright © 2009 Pearson Education, Inc.
•The SI Unit of Magnetic flux = Weber
This figure shows the variables in the flux equation:
ΦB = BA = BA cosθ

Copyright © 2009 Pearson Education, Inc.


•Magnetic Flux is analogous to electric flux:
It
is proportional to the total number of
magnetic field lines passing through the loop.

Copyright © 2009 Pearson Education, Inc.


Faraday’s Law of Induction
Faraday’s Law: The average emf in a circuit (with N
loops) equals the rate of change of magnetic flux
through the circuit:

  f  i
 N  N
t t f  ti

Copyright © 2009 Pearson Education, Inc.


Faraday’s Law of Induction:
“The emf induced in a circuit is equal to the time rate of
change of magnetic flux through the circuit.”

For a coil of N turns:

Copyright © 2009 Pearson Education, Inc.


Faraday’s Law of Induction
 The magnitude of the emf induced in a conducting loop is equal to the rate at which
the magnetic flux through that loop changes with time ,

 If a coil consists of N loops with the same area,


the total induced emf in the coil is given by

 In uniform magnetic field, the induced emf can be


expressed as

150
Example 1: EMF in a loop
A wire loop of radius 0.30m lies so that an external magnetic field
of strength +0.30T is perpendicular to the loop. The field changes
to -0.20T in 1.5s. (The plus and minus signs here refer to opposite
directions through the loop.) Find the magnitude of the average
induced emf in the loop during this time.


B

151
The loop is always perpendicular to the field, so the normal to the loop
is parallel to the field, so cos  = 1. The flux is then

  BA  B r 2

Initially the flux is

 i   0.30T    0.30m  =0.085 T  m 2


2

and after the field changes the flux is

 f   0.20T    0.30m  =-0.057 T  m 2


2

The magnitude of the average induced emf is:

  f   i 0.085 T  m 2 -0.057 T  m 2
emf     0.095V
t t 1.5s

152
Example: One way to Induce an emf in a coil
A coil consists of 200 turns of wire having a total resistance of 2.0
. Each turn is a square of side 18 cm, and a uniform magnetic
field directed perpendicular to the plane of the coil is turned on. If
the field changes linearly form 0 to 0.50 T in 0.80s, what is the
magnitude of the induced emf in the coil while the field is
changing.
 The area of one turn of the coil is (0.18m)2 = 0.0324 m2.
 The magnetic flux through the coil at t=0 is zero because B=0 at that time.
 At t=0.80s, the magnetic flux through one turn is :
B = BA = (0.50T)(0.0324m2) = 0.0162T.m2
 Therefore, the magnitude of the induced emf is :

N B 200(0.0162T.m 2  0T.m 2 )


   4.1T.m 2 / s  4.1V
t 0.80s
153
To induce an emf we can change,

• the magnitude of B
• the area enclosed by the loop
• the angle between B and the normal to the
area
• any combination of the above over time.

154
Sample Problem
Figure shows a conducting loop consisting of a half-circle of radius r=0.20m and
three straight sections. The half-circle lies in a uniform magnetic field that is
directed out of the page; the field magnitude is given by B=4.0t2+2.0 t+3.0,
with B in Tesla's and t in seconds. An ideal battery with emf εbet=2.0V is
connected to the loop. The resistance of the loop is 2.0Ω.
(a) What are the magnitude and direction of the emf induced around the loop by
B field at t=10 s?
b) What is the current in the loop at t=10 s?
Copyright © 2009 Pearson Education, Inc.
•The minus sign gives the
direction of
the induced emf.
 Lenz’s Law:
An induced emf is always
in a direction that OPPOSES
the original change in flux
that caused it.

Copyright © 2009 Pearson Education, Inc.


It is as if the induced emf is producing a virtual bar magnet to oppose
the motion of the bar magnet that is causing the initial changes.
Motion of the magnet increases the flux through the loop pointing to
the right.

The induced current creates a B-field pointing to the left, i.e. a North
pole, to decrease the increased flux.
It is as if the original bar magnet sees an “image” of itself opposing it.
Copyright © 2009 Pearson Education, Inc.
The B-field increases to the right

Copyright © 2009 Pearson Education, Inc.


Copyright © 2009 Pearson Education, Inc.
Copyright © 2009 Pearson Education, Inc.
Copyright © 2009 Pearson Education, Inc.
Copyright © 2009 Pearson Education, Inc.
Copyright © 2009 Pearson Education, Inc.
Copyright © 2009 Pearson Education, Inc.
Example: Lenz’s law
The direction of any magnetic induction effect is such as to oppose the cause
of the effect
For instance: a current produced by an induced emf moves in a direction so
that its magnetic field opposes the original change in flux

S S N
      
N

v N B v B v B v
N S S B
I I I I

Example: If a North pole moves toward the loop in the plane of the page,
in what direction is the induced current?

Since the magnet is moving parallel to the loop,


there is no magnetic flux through the loop.
Thus the induced current is zero.
167
Example 1: A wire loop is being pulled away from a current-carrying wire.
What is the direction of the induced current in the loop?
1) Clockwise
2) Counterclockwise
3) No induced current

On the right side of the wire the magnetic flux is into the page
and decreasing due to the fact that the loop is being pulled
away. By Lenz’s Law, the induced B field will oppose this I
decrease. Thus, the new B field points into the page, which
requires an induced clockwise current to produce such a B field.

Example 2: What is the induced current if the wire loop moves down?

1) Clockwise
2) Counterclockwise
3) No induced current

The magnetic flux through the loop is not changing as it moves


parallel to the wire. Therefore, there is no induced current.
I
168
Example1: A wire loop is being pulled through a uniform magnetic field.
What is the direction of the induced current?
1) Clockwise; 2) Counterclockwise; 3) No induced current x x x x x x x x x
Since the magnetic field is uniform, the magnetic flux through x x x x x x x x x
the loop is not changing. Thus no current is induced. x x x x x x x x x
x x x x x x x x x
Example2: What is the direction of the induced current if the
B field suddenly increases while the loop is in the region? x x x x x x x x x
1) Clockwise 2) Counterclockwise 3) No induced current

The increasing B field into the page must be countered by an induced flux out of the
page. This can be accomplished by induced current in the counterclockwise direction in
the wire loop.

Example 3: A wire loop is being pulled through x x x x x


a uniform magnetic field that suddenly ends.
What is the direction of the induced current? x x x x x
1) Clockwise 2) Counterclockwise 3) No induced current x x x x x
The B field into the page is disappearing in the loop, so x x x x x
it must be compensated by an induced flux also into the
page. This can be accomplished by an induced current
x x x x x
in the clockwise direction in the wire loop. 169
Example : An Exponentially Decaying B
Field
A loop of wire enclosing an area A is
placed in a region where the magnetic field
is perpendicular to the plane of the loop.
The magnitude of B varies in time
according to the expression B = Bmax e-at,
where a is some constant. That is, at t=0
the field is Bmax, and for t>0, the field
decreases exponentially. Find the induced
emf in the loop as a function of time.

170
Solution
 Because B is perpendicular to the plane of the loop, the
magnetic flux thought the loop at time t>0 is : B = BAcos0 =
ABmaxe-at

 Because ABmax and a are constants, the induced emf is :


d B d
  ABmax e at  aABmax e at
dt dt

 This expression indicates that the induced emf decays


exponentially in time.
 Note that the maximum emf occurs at t = 0, where max =
aABmax.

171
Example:
Example : A 12.0-cm-diameter wire coil is initially oriented perpendicular to a 1.5
T magnetic field. The loop is rotated so that its plane is parallel to the field
direction in 0.20 s. What is the average induced emf in the loop?

N 1  B  BA cos 
 f   0  
 B   Bf    Bin   BA cos 0  cos 90  BA
 in   90  B BA
  
2r  12.0cm t t
A  r 2
B  1.5T

1.5T    0.12m / 2 
2
 8.5 10  2 V
t  0.20 s 0.20 s
 ?

173
Applications of Faraday’s law
Rotating loop:   t
  2f  2 T I
 B  BA cos   BA cos t B

 B BA cos t 
   I
t t

  BA sin t  max


Example:
N  200  max  NBA
B  0.03T
A  100cm 2
  100s 1
 max  ?  max  200  0.03T   100  10 4 m 2  100s 1   6V 174
Electric Generator (Dynamo)
•• An electric generator transforms mechanical
energy into electrical energy
•• It consists of many coils of wires wound on
an armature that can be rotated in a magnetic
field
•• An emf is induced in the rotating coil
•• Electric current is the output of a generator
1.• Thus a generator produces alternating current

175
Simple a.c. Generator

• According to the Faraday’s law of


electromagnetic induction,
Nd d
    N BA cos t  NBA sin t
dt dt http://www.walter-fendt.de/ph11e/generator_e.htm
Example: A generator rotates at 60 Hz in a magnetic field of 0.03
T. It has 1000 turns and produces voltage that is 120 V at a pick.
What is the area of each turn of the coil?

f  60 Hz  max  NBA  max


A
B  0.03T   2f 2fNB
N  1000
 max  120V
A? 120V
A  10  2 m 2
2  60 Hz 1000  0.03T 

177
178
Faraday’s disk dynamo

   BR 2
R R
   v  B  dr  B  rdr 
0 0
2
v  r
-

F’m +
Fm v
Motional emf

180
Motional EMF
As the wire moves,

FB  qv  B

Which sets the positive charges in motion in


the direction of FB and leaves negative
charges behind.
As they accumulate on the bottom, an electric field is set up inside.

In equilibrium,

FB  FE or qvB  qE or E  vB

V  El  Blv
181
Motional EMF in a Circuit
 B  BA  Blx

d B d dx
E    Blx    Bl
dt dt dt

E Blv
E   Blv I 
R R

If the bar is moved with constant velocity,

Fapp  FB  IlB

B 2l 2 v 2 E 2
P  Fapp v  IlB v  
R R

182
183
example
A rectangular wire loop is pulled thru a uniform B field
penetrating its top half, as shown. The induced current :
1. Current CW Bout
2. Current CW,
3. Current CCW,
4. Current CCW, v
5. No current,
. No current
The motion does not change the magnetic flux, so Faraday’s Law says there is no
induced EMF, or current, or force, or torque.
Of course, if we were pulling at all up or down there would be a force to oppose
that motion.
example
A circuit in the form of a rectangular piece of
wire is pulled away from a long wire carrying
current I in the direction shown in the sketch.
The induced current in the rectangular circuit is
1. Clockwise
2. Counterclockwise
3. Neither, the current is zero
1. Induced current is clockwise

•B due to I is into page; the flux through the circuit due to that
field decreases as the circuit moves away. So the induced
current is clockwise (to make a B into the page)
Example
Consider the circuit .the length of the moving rod is 0.2 m, its speed
is 0.1m/s , the magnetic field-strength is 1T), and the resistance of
the circuit is 0.02Ω)

1.What is the emf generated around the circuit?

2.What current flows around the circuit?


3.What is the magnitude and direction of the force acting on the
moving rod due to the fact that a current is flowing along it?

F  ILB  1 1 0.2  0.2 N

4.What is the power delivered by the applied force?


P  Fv  0.2  0.1  0.02W
P  I  I 2 R
Problem
A potential difference of 0.050 V is developed across a
10-cm long wire as it moves through a magnetic field at
5.0 m/s. The magnetic field is perpendicular to the wire
axis. What are strength and direction of field?

188
Problem (ans)
At equilibrium
Fm Fe

Out of page

189
Example : Magnetic Force Acting on a Sliding Bar
The conducting bar illustrated in Figure, of mass m and  length ,
moves on two frictionless parallel rails in the presence of a uniform
magnetic field directed into the page. The bar is given an initial
velocity vi to the right and is released at t=0. Find the velocity of
the bar as a function of time.

190
Solution
 The induced current is counterclockwise, and the magnetic force is
FB = -I  B, where the negative sign denotes that the force is to the left and
retards the motion.
 This is the only horizontal force acting on the bar, and hence Newton’s second
law applied to motion in the horizontal direction gives :
dv
Fx  ma  m   IB
dt

 We know that I = Bv/R, and so we can write this expression as :


dv B2 2
m  v
dt R

dv  B2 2 
  dt
v  mR 

191
 Integrating this equation using the initial condition that v=vi at t=0, we find
that:
vdv  B 2 2 t
vi
 0 dt
v mR
v  B 2 2  t
ln     t  
 vi   mR  

mR
 where the constant  B 2 2

 From this result, we


t / 
see that the velocity can be expressed in the exponential
form : v  v i e
 This expression indicates that the velocity of the bar decreases exponentially
with time under the action of the magnetic retarding force.

192
193
194
Figure shows a right view of a bar that can slide ]4[
without friction. The resistor is 6.00 Ω and a 2.50T
magnetic field is directed perpendicularly downward,
.into the paper. Let L= 1.20 m
Calculate the applied force required to move the )a(
.bar to the right at a constant speed of 2.00 m/s
?At what rate is energy delivered to the resistor )b(

195
196
197
Induced nonelectrostatic electric field
d B
dI
 
dt
0 dt
Origin of emf? No motion, moreover no B outside
solenoid, i.e. in the region of a wire loop. Then it
should be E which results in induced current.

 
Fel  dl d B
 q     dt
dI    
0 Fel  qE ,  Fel  dl  0 Nonconcervative force
dt
  d B
 E  dl   dt
 
 E  d l  0 Nonelectrostatic field
B(t) should induce E by independently on the presence of the loop of the wire!
Let’s find E(r).

  d B
 E  dl   dt
1.r  R
dB
R E 2r  r 2

dt
B  ni,   K m  0
n di
E r
2 dt
E
2) r  R
dB
E 2r  R 2

dt
r
n di R 2
R E
2 dt r
Example : Electric Field Induced by a Changing Magnetic
Field in a Solenoid
A long solenoid of radius R has n turns of wire per unit length
and carries a time-varying current that varies sinusoidally as:
I = Imax cos t, where Imax is the maximum current and  is the
angular frequency of the alternating current source Figure. (a)
Determine the magnitude of the induced electric field outside the
solenoid, a distance r > R from its long central axis. (b) What is
the magnitude of the induced elelctric field inside the solenoid, a
distance r from its axis?

200
Solution for (a)
Consider an external point and take the path for our line integral to be a circle
of radius r centered on the solenoid as illustrated in Figure (31.18).
The magnitude of E is constant on this path and that E is tangent to it.
The magnetic flux through the area enclosed by this path is BA = B R2; hence :
  d 2 dB
      
2
E ds ( B R ) R
dt dt
  2 dB (1)
 E  ds  E (2r )  R dt
The magnetic field inside a long solenoid is given by , B = onI.

Substitute I = Imax cos t into this equation and then substitute the result into
Eq. (1), we find that :

201
d
E (2r )   R 2 o nI max (cos t )
dt
 R 2 o nI max  sin t

 o nI max R 2
E sin t (for r > R) (2)
2r

202
Solution for (b)
 For an interior point (r < R), the flux threading an integration loop is given
by Br2.
 Using the same procedure as in part (a), we find that :

dB
E (2r )   r 2  r 2 o nI max  sin t
dt

 o nI max  (3) (for r < R)


E r sin t
2

203
Example: An AC generator consists of 8 turns of wire,
each of area A = 0.090 0 m2 , and the total resistance of
the wire is 12.0 (. The loop rotates in a 0.500-T magnetic
.field at a constant frequency of 60.0 Hz
.Find the maximum induced emf )A(
  2f  377 s 1
 max  NAB  136V

What is the maximum induced current when the output )B(


?terminals are connected to a low-resistance conductor
 max
I max   11 .3 A
R

204
Maxwell’s Equations
Q
 E.dA   0
Gauss’ Law

Gauss’ Law for Magnetism


 B.dA  0 no magnetic monopoles

d B Faraday’s Law
 E.ds   dt
d E
 B.ds  0 I  0 0 dt Ampère-Maxwell Law

Lorentz Force Law


F  qE  qv  B

205
A rectangular coil of 150 loops forms a ]1[
closed circuit with a resistance of 5 and
measures 0.2 m wide by 0.1 m deep, as
shown below. The circuit is placed between
the poles of an electromagnet which is
producing a uniform magnetic field of 40 T.
The magnet is switched off, causing the
magnetic field to drop to zero in 2 s. (The
loops are parallel with the faces of the
electromagnet.)
a) Compute the average induced potential in the
.circuit
.b) Determine the average current in the circuit
c) Indicate on the drawing the direction in which the
.induced current flows

206
a ]   (0.1)(0.2)[0  40]  0.8T .m 2

d  0.8
  N  (150)[ ]  60V
dt 2
V 60
b] I    12 A
R 5

C] The magnetic field from the magnet is up, but the flux
is decreasing. So the magnetic field from the current
induced in the loop(s) will also be up. Thus the current
.flow is left-to-right across the front of the loop

207
[2] A conducting rod of length l moves on two (frictionless)
horizontal rails, as shown to the right. A constant force of
magnitude
 
Fapp  1.0 N moves the bar at a constant speed of v  2.0m / s

through a magnetic field B
R  8.0
directed into the page. The resistor has a value

( a) What is the current through the resistor


R?
(b) What is the mechanical power
delivered by the
constant force?

208
When the conducting rod moves to the right, this serves to increase the flux as
time passes , so any induced current wants to stop this change and decrease the
magnetic flux. Therefore, the induced current will act in such a way to oppose
the external field (i.e., the field due to the induced current will be opposite to
.the external field). This must be a counterclockwise current

To find the current, we only need to find the motionally-induced voltage and
then apply Ohm’s law. The rod is just a bar of length l moving at velocity v in a
:magnetic field B. This gives us an voltage ∆V , and Ohm’s law gives us I

Blv
ΔV  Blv  IR  I
R

209
Keep in mind that v is velocity, while ∆V is voltage. Of course, the problem
is now that we don’t know B. We do know that the external and magnetic
forces must balance for the rod to have a constant velocity however.
Constant velocity implies zero acceleration, which implies no net force. If
this is to be true, the applied force must exactly balance the magnetic force
on the rod. For a conductor of length l carrying a current I in a field B, we
know how to calculate the magnetic force

Fapp
Fm  BIl  Fapp  B
Il
:Plug that into the first equation

V Blv Fapp lv Fapp v Fapp v


I     I 2

R R IlR IR R

.You should get I=0.5 A


210
What about the power? Conservation of energy tells us that the mechanical
power delivered must be the same as the power dissipated in the resistor, or
PF  Fv cos 
.You should get 2 W

Alternatively, you note that power delivered by a force is

PF  PR  I 2 R

where θ is the angle between the force and velocity. In this case, θ = 0, so

PF  Fv  2W

211

You might also like